LSAT and Law School Admissions Forum

Get expert LSAT preparation and law school admissions advice from PowerScore Test Preparation.

 Administrator
PowerScore Staff
  • PowerScore Staff
  • Posts: 8917
  • Joined: Feb 02, 2011
|
#43372
Please post your questions below! Thank you!
 Michaeltinti22
  • Posts: 12
  • Joined: Sep 04, 2018
|
#60029
So I chose E because I focused on the aspect that Ted makes irreplaceable contributions in spite of not working as long as others; at the time, I saw E as acknowledging that since time isn't the only consideration to take into account, it would be wrong to solely fire someone based on the time they spend in work. Now, I see that A is correct, in that it presents a much firmer conditional logic chain. We know that this situation doesn't satisfy the necessary condition, so if A is correct then the supervisor shouldn't request for Ted to be replaced. I'm just curious if there's anything about E that also makes it a bad answer, other than A being clearly stronger.
 Adam Tyson
PowerScore Staff
  • PowerScore Staff
  • Posts: 5153
  • Joined: Apr 14, 2011
|
#60741
Good question, and good analysis, Michaeltinti22! The biggest problem with answer E is that it fails to address the crucial issue of whether Tatiana should request that Ted be replaced. So what if Ted's contributions are not always a function of his time? He's still working short hours, and his coworkers are still having to make up for his doing less work than he should. This rule does nothing, by itself and without helping it with outside information, to help support Tatiana's decision. In order for E to strengthen her decision, we would have to add that Ted's contributions compensate for his shortcomings, and that is outside the scope of what we were given.

Put another way, to strengthen a decision not to replace him, we need a principle that talks about replacing people. Anything that fails to directly address that in some way will fall short of supporting that conclusion.

I hope that helps clear it up! Keep up the good work!
 JiminyC
  • Posts: 5
  • Joined: Jan 09, 2019
|
#61849
my problem with A is that what if the employee does absolutely nothing, and yet the other employees are still working harder than they should to make up for the work? Under this principle, Tatiana should still not request to replace Ted.
 Robert Carroll
PowerScore Staff
  • PowerScore Staff
  • Posts: 1787
  • Joined: Dec 06, 2013
|
#61869
Jiminy,

That's not a problem with answer choice (A). You ultimately want to strengthen the case for Tatiana not to replace Ted. If answer choice (A) would bolster that case even in some seemingly extreme situations, that's actually great for the answer! It means it provides a very strong basis for not requesting his replacement, and since we want to strengthen that case, that's a good thing. If this were an Assumption question, an issue might arise that answer choice (A) is overly broad and thus doesn't need to be assumed. But in a Strengthen or Justify scenario, broad applicability is perfectly fine.

Robert Carroll
User avatar
 cornflakes
  • Posts: 48
  • Joined: Feb 19, 2021
|
#87462
Hi Powerscore,

Went with C instead of A on my PT - on my BR it took me a few mins, but noticed that C seems to be a shell game of the stim - "decide whether that employee should be replaced" as opposed to "request that Ted (an employee) be replaced.

Is this the reason why C is incorrect?
 Adam Tyson
PowerScore Staff
  • PowerScore Staff
  • Posts: 5153
  • Joined: Apr 14, 2011
|
#87861
Correct! This isn't about whether Tatiana has any authority to decide Ted's fate, but about her not even asking whoever is in charge to fire him. That's the key to eliminate answer C!

Get the most out of your LSAT Prep Plus subscription.

Analyze and track your performance with our Testing and Analytics Package.